Medical Surgical Nursing

Published on July 2016 | Categories: Documents | Downloads: 30 | Comments: 0 | Views: 272
of 35
Download PDF   Embed   Report

Comments

Content

MEDICAL SURGICAL NURSING
1. Following surgery, Mario complains of mild incisional pain while performing deep- breathing and coughing exercises. The nurse’s best response would be: A.“Pain will become less each day.” B.“This is a normal reaction after surgery.” C.“With a pillow, apply pressure against the incision.” D.“I will give you the pain medication the physician ordered.” Answer: (C) “With a pillow, apply pressure against the incision.” Applying pressure against the incision with a pillow will help lessen the intra-abdominal pressure created by coughing which causes tension on the incision that leads to pain. 2. The nurse needs to carefully assess the complaint of pain of the elderly because older people A.are expected to experience chronic pain B.have a decreased pain threshold C.experience reduced sensory perception D.have altered mental function Answer: (C) experience reduced sensory perception Degenerative changes occur in the elderly. The response to pain in the elderly maybe lessened because of reduced acuity of touch, alterations in neural pathways and diminished processing of sensory data. 3. Mary received AtropineSO4 as a pre-medication 30 minutes ago and is now complaining of dry mouth and her PR is higher, than before the medication was administered. The nurse’s best A.The patient is having an allergic reaction to the drug. B.The patient needs a higher dose of this drug C.This is normal side-effect of AtSO4 D.The patient is anxious about upcoming surgery Answer: (C) This is normal side-effect of AtSO4 Atropine sulfate is a vagolytic drug that decreases oropharyngeal secretions and increases the heart rate. 4. Ana’s postoperative vital signs are a blood pressure of 80/50 mm Hg, a pulse of 140, and respirations of 32. Suspecting shock, which of the following orders would the nurse question? A.Put the client in modified Trendelenberg's position. B.Administer oxygen at 100%. C.Monitor urine output every hour. D.Administer Demerol 50mg IM q4h Answer: (D) Administer Demerol 50mg IM q4h Administering Demerol, which is a narcotic analgesic, can depress respiratory and cardiac function and thus not given to a patient in shock. What is needed is promotion for adequate oxygenation and perfusion. All the other interventions can be expected to be done by the nurse. 5. Mr. Pablo, diagnosed with Bladder Cancer, is scheduled for a cystectomy with the creation of an ileal conduit in the morning. He is wringing his hands and pacing the floor when the nurse enters his room. What is the best approach? A."Good evening, Mr. Pablo. Wasn't it a pleasant day, today?" B."Mr, Pablo, you must be so worried, I'll leave you alone with your thoughts. C.“Mr. Pablo, you'll wear out the hospital floors and yourself at this rate." D."Mr. Pablo, you appear anxious to me. How are you feeling about tomorrow's surgery?" Answer: (D) "Mr. Pablo, you appear anxious to me. How are you feeling about tomorrow's surgery?" The client is showing signs of anxiety reaction to a stressful event. Recognizing the client’s anxiety conveys acceptance of his behavior and will allow for verbalization of feelings and concerns. 6. After surgery, Gina returns from the Post-anesthesia Care Unit (Recovery Room) with a nasogastric tube in place following a gall bladder surgery. She continues to complain of nausea. Which action would the nurse take? A.Call the physician immediately. B.Administer the prescribed antiemetic. C.Check the patency of the nasogastric tube for any obstruction. D.Change the patient’s position.

Answer: (C) Check the patency of the nasogastric tube for any obstruction. Nausea is one of the common complaints of a patient after receiving general anesthesia. But this complaint could be aggravated by gastric distention especially in a patient who has undergone abdominal surgery. Insertion of the NGT helps relieve the problem. Checking on the patency of the NGT for any obstruction will help the nurse determine the cause of the problem and institute the necessary intervention. 7. Mr. Perez is in continuous pain from cancer that has metastasized to the bone. Pain medication provides little relief and he refuses to move. The nurse should plan to: A.Reassure him that the nurses will not hurt him B.Let him perform his own activities of daily living C.Handle him gently when assisting with required care D.Complete A.M. care quickly as possible when necessary Answer: (C) Handle him gently when assisting with required care Patients with cancer and bone metastasis experience severe pain especially when moving. Bone tumors weaken the bone to appoint at which normal activities and even position changes can lead to fracture. During nursing care, the patient needs to be supported and handled gently. 8. A client returns from the recovery room at 9AM alert and oriented, with an IV infusing. His pulse is 82, blood pressure is 120/80, respirations are 20, and all are within normal range. At 10 am and at 11 am, his vital signs are stable. At noon, however, his pulse rate is 94, blood pressure is 116/74, and respirations are 24. What nursing action is most appropriate? A.Notify his physician. B.Take his vital signs again in 15 minutes. C.Take his vital signs again in an hour. D.Place the patient in shock position. Answer: (B) Take his vital signs again in 15 minutes. Monitoring the client’s vital signs following surgery gives the nurse a sound information about the client’s condition. Complications can occur during this period as a result of the surgery or the anesthesia or both. Keeping close track of changes in the VS and validating them will help the nurse initiate interventions to prevent complications from occurring. 9. A 56 year old construction worker is brought to the hospital unconscious after falling from a 2-story building. When assessing the client, the nurse would be most concerned if the assessment revealed: A.Reactive pupils B.A depressed fontanel C.Bleeding from ears D.An elevated temperature Answer: (C) Bleeding from ears The nurse needs to perform a thorough assessment that could indicate alterations in cerebral function, increased intracranial pressures, fractures and bleeding. Bleeding from the ears occurs only with basal skull fractures that can easily contribute to increased intracranial pressure and brain herniation 10. Which of the ff. statements by the client to the nurse indicates a risk factor for CAD? A.“I exercise every other day.” B.“My father died of Myasthenia Gravis.” C.“My cholesterol is 180.” D.“I smoke 1 1/2 packs of cigarettes per day.” Answer: (D) “I smoke 1 1/2 packs of cigarettes per day.” Smoking has been considered as one of the major modifiable risk factors for coronary artery disease. Exercise and maintaining normal serum cholesterol levels help in its prevention. 11. Mr. Braga was ordered Digoxin 0.25 mg. OD. Which is poor knowledge regarding this drug? A.It has positive inotropic and negative chronotropic effects B.The positive inotropic effect will decrease urine output C.Toxixity can occur more easily in the presence of hypokalemia, liver and renal problems

D.Do not give the drug if the apical rate is less than 60 beats per minute. Answer: (B) The positive inotropic effect will decrease urine output Inotropic effect of drugs on the heart causes increase force of its contraction. This increases cardiac output that improves renal perfusion resulting in an improved urine output. 12. Valsalva maneuver can result in bradycardia. Which of the following activities will not stimulate Valsalva's maneuver? A.Use of stool softeners. B. Enema administration C.Gagging while toothbrushing. D.Lifting heavy objects Answer: (A) Use of stool softeners. Straining or bearing down activities can cause vagal stimulation that leads to bradycardia. Use of stool softeners promote easy bowel evacuation that prevents straining or the valsalva maneuver. 13. The nurse is teaching the patient regarding his permanent artificial pacemaker. Which information given by the nurse shows her knowledge deficit about the artificial cardiac pacemaker? A.take the pulse rate once a day, in the morning upon awakening B.may be allowed to use electrical appliances C.have regular follow up care D.may engage in contact sports Answer: (D) may engage in contact sports The client should be advised by the nurse to avoid contact sports. This will prevent trauma to the area of the pacemaker generator. 14. A patient with angina pectoris is being discharged home with nitroglycerine tablets. Which of the following instructions does the nurse include in the teaching? A.“When your chest pain begins, lie down, and place one tablet under your tongue. If the pain continues, take another tablet in 5 minutes.” B.“Place one tablet under your tongue. If the pain is not relieved in 15 minutes, go to the hospital.” C.“Continue your activity, and if the pain does not go away in 10 minutes, begin taking the nitro tablets one every 5 minutes for 15 minutes, then go lie down.” D. “Place one Nitroglycerine tablet under the tongue every five minutes for three doses. Go to the hospital if the pain is unrelieved. Answer: (D) “Place one Nitroglycerine tablet under the tongue every five minutes for three doses. Go to the hospital if the pain is unrelieved. Angina pectoris is caused by myocardial ischemia related to decreased coronary blood supply. Giving nitroglycerine will produce coronary vasodilation that improves the coronary blood flow in 3 – 5 mins. If the chest pain is unrelieved, after three tablets, there is a possibility of acute coronary occlusion that requires immediate medical attention. 15. A client with chronic heart failure has been placed on a diet restricted to 2000mg. of sodium per day. The client demonstrates adequate knowledge if behaviors are evident such as not salting food and avoidance of which food? A.Whole milk B.Canned sardines C.Plain nuts D.Eggs Answer: (B) Canned sardines Canned foods are generally rich in sodium content as salt is used as the main preservative. 16. A student nurse is assigned to a client who has a diagnosis of thrombophlebitis. Which action by this team member is most appropriate? A.Apply a heating pad to the involved site. B.Elevate the client's legs 90 degrees. C.Instruct the client about the need for bed rest.

D.Provide active range-of-motion exercises to both legs at least twice every shift. Answer: (C) Instruct the client about the need for bed rest. In a client with thrombophlebitis, bedrest will prevent the dislodgment of the clot in the extremity which can lead to pulmonary embolism. 17. A client receiving heparin sodium asks the nurse how the drug works. Which of the following points would the nurse include in the explanation to the client? A.It dissolves existing thrombi. B.It prevents conversion of factors that are needed in the formation of clots. C.It inactivates thrombin that forms and dissolves existing thrombi. D.It interferes with vitamin K absorption. Answer: (B) It prevents conversion of factors that are needed in the formation of clots. Heparin is an anticoagulant. It prevents the conversion of prothrombin to thrombin. It does not dissolve a clot. 18. The nurse is conducting an education session for a group of smokers in a “stop smoking” class. Which finding would the nurse state as a common symptom of lung cancer? : A.Dyspnea on exertion B.Foamy, blood-tinged sputum C.Wheezing sound on inspiration D.Cough or change in a chronic cough Answer: (D) Cough or change in a chronic cough Cigarette smoke is a carcinogen that irritates and damages the respiratory epithelium. The irritation causes the cough which initially maybe dry, persistent and unproductive. As the tumor enlarges, obstruction of the airways occurs and the cough may become productive due to infection. 19. Which is the most relevant knowledge about oxygen administration to a client with COPD? A.Oxygen at 1-2L/min is given to maintain the hypoxic stimulus for breathing. B.Hypoxia stimulates the central chemoreceptors in the medulla that makes the client breath. C.Oxygen is administered best using a non-rebreathing mask D.Blood gases are monitored using a pulse oximeter. Answer: (A) Oxygen at 1-2L/min is given to maintain the hypoxic stimulus for breathing. COPD causes a chronic CO2 retention that renders the medulla insensitive to the CO2 stimulation for breathing. The hypoxic state of the client then becomes the stimulus for breathing. Giving the clientoxygen in low concentrations will maintain the client’s hypoxic drive. 20. When suctioning mucus from a client's lungs, which nursing action would be least appropriate? A.Lubricate the catheter tip with sterile saline before insertion. B.Use sterile technique with a two-gloved approach C.Suction until the client indicates to stop or no longer than 20 second D.Hyperoxygenate the client before and after suctioning Answer: (C) Suction until the client indicates to stop or no longer than 20 second One hazard encountered when suctioning a client is the development of hypoxia. Suctioning sucks not only the secretions but also the gases found in the airways. This can be prevented by suctioning the client for an average time of 5-10 seconds and not more than 15 seconds and hyperoxygenating the client before and after suctioning. 21. Dr. Santos prescribes oral rifampin (Rimactane) and isoniazid (INH) for a client with a positive Tuberculin skin test. When informing the client of this decision, the nurse knows that the purpose of this choice of treatment is to A.Cause less irritation to the gastrointestinal tract B.Destroy resistant organisms and promote proper blood levels of the drugs C.Gain a more rapid systemic effect D.Delay resistance and increase the tuberculostatic effect Answer: (D) Delay resistance and increase the tuberculostatic effect Pulmonary TB is treated primarily with chemotherapeutic agents for 6-12 mons. A prolonged treatment duration is necessary to ensure eradication of the organisms and to prevent relapse. The increasing prevalence of drug

resistance points to the need to begin the treatment with drugs in combination. Using drugs in combination can delay the drug resistance. 22. Mario undergoes a left thoracotomy and a partial pneumonectomy. Chest tubes are inserted, and one-bottle water-seal drainage is instituted in the operating room. In the postanesthesia care unit Mario is placed in Fowler's position on either his right side or on his back to A.Reduce incisional pain. B.Facilitate ventilation of the left lung. C.Equalize pressure in the pleural space. D.Increase venous return Answer: (B) Facilitate ventilation of the left lung. Since only a partial pneumonectomy is done, there is a need to promote expansion of this remaining Left lung by positioning the client on the opposite unoperated side. 23. A client with COPD is being prepared for discharge. The following are relevant instructions to the client regarding the use of an oral inhaler EXCEPT A.Breath in and out as fully as possible before placing the mouthpiece inside the mouth. B.Inhale slowly through the mouth as the canister is pressed down C.Hold his breath for about 10 seconds before exhaling D.Slowly breath out through the mouth with pursed lips after inhaling the drug. Answer: (D) Slowly breath out through the mouth with pursed lips after inhaling the drug. If the client breathes out through the mouth with pursed lips, this can easily force the just inhaled drug out of the respiratory tract that will lessen its effectiveness. 24. A client is scheduled for a bronchoscopy. When teaching the client what to expect afterward, the nurse's highest priority of information would be A.Food and fluids will be withheld for at least 2 hours. B.Warm saline gargles will be done q 2h. C.Coughing and deep-breathing exercises will be done q2h. D.Only ice chips and cold liquids will be allowed initially. Answer: (A) Food and fluids will be withheld for at least 2 hours. Prior to bronchoscopy, the doctors sprays the back of the throat with anesthetic to minimize the gag reflex and thus facilitate the insertion of the bronchoscope. Giving the client food and drink after the procedure without checking on the return of the gag reflex can cause the client to aspirate. The gag reflex usually returns after two hours. 25. The nurse enters the room of a client with chronic obstructive pulmonary disease. The client's nasal cannula oxygen is running at a rate of 6 L per minute, the skin color is pink, and the respirations are 9 per minute and shallow. What is the nurse’s best initial action? A.Take heart rate and blood pressure. B.Call the physician. C.Lower the oxygen rate. D.Position the client in a Fowler's position. Answer: (C) Lower the oxygen rate. The client with COPD is suffering from chronic CO2 retention. The hypoxic drive is his chief stimulus for breathing. Giving O2 inhalation at a rate that is more than 2-3L/min can make the client lose his hypoxic drive which can be assessed as decreasing RR. 26. The nurse is preparing her plan of care for her patient diagnosed with pneumonia. Which is the most appropriate nursing diagnosis for this patient? A.Fluid volume deficit B.Decreased tissue perfusion. C.Impaired gas exchange. D.Risk for infection Answer: (C) Impaired gas exchange.

Pneumonia, which is an infection, causes lobar consolidation thus impairing gas exchange between the alveoli and the blood. Because the patient would require adequate hydration, this makes him prone to fluid volume excess. 27. A nurse at the weight loss clinic assesses a client who has a large abdomen and a rounded face. Which additional assessment finding would lead the nurse to suspect that the client has Cushing’s syndrome rather than obesity? A. large thighs and upper arms B. pendulous abdomen and large hips C.abdominal striae and ankle enlargement D.posterior neck fat pad and thin extremities Answer: (D) posterior neck fat pad and thin extremities “Buffalo hump” is the accumulation of fat pads over the upper back and neck. Fat may also accumulate on the face. There is truncal obesity but the extremities are thin. All these are noted in a client with Cushing’s syndrome. 28. Which statement by the client indicates understanding of the possible side effects of Prednisone therapy? A.“I should limit my potassium intake because hyperkalemia is a side-effect of this drug.” B.“I must take this medicine exactly as my doctor ordered it. I shouldn’t skip doses.” C.“This medicine will protect me from getting any colds or infection.” D.“My incision will heal much faster because of this drug.” Answer: (B) “I must take this medicine exactly as my doctor ordered it. I shouldn’t skip doses.” The possible side effects of steroid administration are hypokalemia, increase tendency to infection and poor wound healing. Clients on the drug must follow strictly the doctor’s order since skipping the drug can lower the drug level in the blood that can trigger acute adrenal insufficiency or Addisonian Crisis 29. A client, who is suspected of having Pheochromocytoma, complains of sweating, palpitation and headache. Which assessment is essential for the nurse to make first? A.Pupil reaction B.Hand grips C.Blood pressure D.Blood glucose Answer: (C) Blood pressure Pheochromocytoma is a tumor of the adrenal medulla that causes an increase secretion of catecholamines that can elevate the blood pressure. 30. The nurse is attending a bridal shower for a friend when another guest, who happens to be a diabetic, starts to tremble and complains of dizziness. The next best action for the nurse to take is to: A.Encourage the guest to eat some baked macaroni B.Call the guest’s personal physician C.Offer the guest a cup of coffee D.Give the guest a glass of orange juice Answer: (D) Give the guest a glass of orange juice In diabetic patients, the nurse should watch out for signs of hypoglycemia manifested by dizziness, tremors, weakness, pallor diaphoresis and tachycardia. When this occurs in a conscious client, he should be given immediately carbohydrates in the form of fruit juice, hard candy, honey or, if unconscious, glucagons or dextrose per IV. 31. An adult, who is newly diagnosed with Graves disease, asks the nurse, “Why do I need to take Propanolol (Inderal)?” Based on the nurse’s understanding of the medication and Grave’s disease, the best response would be: A.“The medication will limit thyroid hormone secretion.” B.“The medication limit synthesis of the thyroid hormones.” C.“The medication will block the cardiovascular symptoms of Grave’s disease.” D.“The medication will increase the synthesis of thyroid hormones.” Answer: (C) “The medication will block the cardiovascular symptoms of Grave’s disease.” Propranolol (Inderal) is a beta-adrenergic blocker that controls the cardiovascular manifestations brought about by increased secretion of the thyroid hormone in Grave’s disease.

32. During the first 24 hours after thyroid surgery, the nurse should include in her care: A.Checking the back and sides of the operative dressing B.Supporting the head during mild range of motion exercise C.Encouraging the client to ventilate her feelings about the surgery D.Advising the client that she can resume her normal activities immediately Answer: (A) Checking the back and sides of the operative dressing Following surgery of the thyroid gland, bleeding is a potential complication. This can best be assessed by checking the back and the sides of the operative dressing as the blood may flow towards the side and back leaving the front dry and clear of drainage. 33. On discharge, the nurse teaches the patient to observe for signs of surgically induced hypothyroidism. The nurse would know that the patient understands the teaching when she states she should notify the MD if she develops: A.Intolerance to heat B.Dry skin and fatigue C.Progressive weight gain D.Insomnia and excitability Answer: (C) Progressive weight gain Hypothyroidism, a decrease in thyroid hormone production, is characterized by hypometabolism that manifests itself with weight gain. 34. What is the best reason for the nurse in instructing the client to rotate injection sites for insulin? A.Lipodystrophy can result and is extremely painful B.Poor rotation technique can cause superficial hemorrhaging C.Lipodystrophic areas can result, causing erratic insulin absorption rates from these D.Injection sites can never be reused Answer: (C) Lipodystrophic areas can result, causing erratic insulin absorption rates from these Lipodystrophy is the development of fibrofatty masses at the injection site caused by repeated use of an injection site. Injecting insulin into these scarred areas can cause the insulin to be poorly absorbed and lead to erratic reactions. 35. Which of the following would be inappropriate to include in a diabetic teaching plan? A.Change position hourly to increase circulation B.Inspect feet and legs daily for any changes C. Keep legs elevated on 2 pillows while sleeping D.Keep the insulin not in use in the refrigerator Answer: (C) Keep legs elevated on 2 pillows while sleeping The client with DM has decreased peripheral circulation caused by microangiopathy. Keeping the legs elevated during sleep will further cause circulatory impairment. 36. Included in the plan of care for the immediate post-gastroscopy period will be: A.Maintain NGT to intermittent suction B.Assess gag reflex prior to administration of fluids C. Assess for pain and medicate as ordered D.Measure abdominal girth every 4 hours Answer: (B) Assess gag reflex prior to administration of fluids The client, after gastroscopy, has temporary impairment of the gag reflex due to the anesthetic that has been sprayed into his throat prior to the procedure. Giving fluids and food at this time can lead to aspiration. 36. Included in the plan of care for the immediate post-gastroscopy period will be: A.Maintain NGT to intermittent suction B.Assess gag reflex prior to administration of fluids C. Assess for pain and medicate as ordered D.Measure abdominal girth every 4 hours Answer: (B) Assess gag reflex prior to administration of fluids

The client, after gastroscopy, has temporary impairment of the gag reflex due to the anesthetic that has been sprayed into his throat prior to the procedure. Giving fluids and food at this time can lead to aspiration. 37. Which description of pain would be most characteristic of a duodenal ulcer? A.Gnawing, dull, aching, hungerlike pain in the epigastric area that is relieved by food intake B.RUQ pain that increases after meal C.Sharp pain in the epigastric area that radiates to the right shoulder D.A sensation of painful pressure in the midsternal area Answer: (A) Gnawing, dull, aching, hungerlike pain in the epigastric area that is relieved by food intake Duodenal ulcer is related to an increase in the secretion of HCl. This can be buffered by food intake thus the relief of the pain that is brought about by food intake. 38. The client underwent Billroth surgery for gastric ulcer. Post-operatively, the drainage from his NGT is thick and the volume of secretions has dramatically reduced in the last 2 hours and the client feels like vomiting. The most appropriate nursing action is to: A. Reposition the NGT by advancing it gently NSS B. Notify the MD of your findings C. Irrigate the NGT with 50 cc of sterile D. Discontinue the low-intermittent suction Answer: (B) Notify the MD of your findings The client’s feeling of vomiting and the reduction in the volume of NGT drainage that is thick are signs of possible abdominal distention caused by obstruction of the NGT. This should be reported immediately to the MD to prevent tension and rupture on the site of anastomosis caused by gastric distention. 39. After Billroth II Surgery, the client developed dumping syndrome. Which of the following should the nurse exclude in the plan of care? A.Sit upright for at least 30 minutes after meals B.Take only sips of H2O between bites of solid food C.Eat small meals every 2-3 hours D.Reduce the amount of simple carbohydrate in the diet Answer: (A) Sit upright for at least 30 minutes after meals The dumping syndrome occurs within 30 mins after a meal due to rapid gastric emptying, causing distention of the duodenum or jejunum produced by a bolus of food. To delay the emptying, the client has to lie down after meals. Sitting up after meals will promote the dumping syndrome. 40. The laboratory of a male patient with Peptic ulcer revealed an elevated titer of Helicobacter pylori. Which of the following statements indicate an understanding of this data? A.Treatment will include Ranitidine and Antibiotics B. No treatment is necessary at this time C.This result indicates gastric cancer caused by the organism D.Surgical treatment is necessary Answer: (A) Treatment will include Ranitidine and Antibiotics One of the causes of peptic ulcer is H. Pylori infection. It releases toxin that destroys the gastric and duodenal mucosa which decreases the gastric epithelium’s resistance to acid digestion. Giving antibiotics will control the infection and Ranitidine, which is a histamine-2 blocker, will reduce acid secretion that can lead to ulcer. 41. What instructions should the client be given before undergoing a paracentesis? A.NPO 12 hours before procedure B. Empty bladder before procedure C.Strict bed rest following procedure D.Empty bowel before procedure Answer: (B) Empty bladder before procedure Paracentesis involves the removal of ascitic fluid from the peritoneal cavity through a puncture made below the

umbilicus. The client needs to void before the procedure to prevent accidental puncture of a distended bladder during the procedure. 42. The husband of a client asks the nurse about the protein-restricted diet ordered because of advanced liver disease. What statement by the nurse would best explain the purpose of the diet? A.“The liver cannot rid the body of ammonia that is made by the breakdown of protein in the digestive system.” B.“The liver heals better with a high carbohydrates diet rather than protein.” C.“Most people have too much protein in their diets. The amount of this diet is better for liver healing.” D.“Because of portal hyperemesis, the blood flows around the liver and ammonia made from protein collects in the brain causing hallucinations.” Answer: (A) “The liver cannot rid the body of ammonia that is made by the breakdown of protein in the digestive system.” The largest source of ammonia is the enzymatic and bacterial digestion of dietary and blood proteins in the GI tract. A protein-restricted diet will therefore decrease ammonia production. 43. Which of the drug of choice for pain controls the patient with acute pancreatitis? A.Morphine B.NSAIDS C.Meperidine D.Codeine Answer: (C) Meperidine Pain in acute pancreatitis is caused by irritation and edema of the inflamed pancreas as well as spasm due to obstruction of the pancreatic ducts. Demerol is the drug of choice because it is less likely to cause spasm of the Sphincter of Oddi unlike Morphine which is spasmogenic. 44. Immediately after cholecystectomy, the nursing action that should assume the highest priority is: A.encouraging the client to take adequate deep breaths by mouth B.encouraging the client to cough and deep breathe C.changing the dressing at least BID D. irrigate the T-tube frequently Answer: (B) encouraging the client to cough and deep breathe Cholecystectomy requires a subcostal incision. To minimize pain, clients have a tendency to take shallow breaths which can lead to respiratory complications like pneumonia and atelectasis. Deep breathing and coughing exercises can help prevent such complications. 45. A Sengstaken-Blakemore tube is inserted in the effort to stop the bleeding esophageal varices in a patient with complicated liver cirrhosis. Upon insertion of the tube, the client complains of difficulty of breathing. The first action of the nurse is to: A. Deflate the esophageal balloon B. Monitor VS C. Encourage him to take deep breaths D. Notify the MD Answer: (A) Deflate the esophageal balloon When a client with a Sengstaken-Blakemore tube develops difficulty of breathing, it means the tube is displaced and the inflated balloon is in the oropharynx causing airway obstruction 46. The client presents with severe rectal bleeding, 16 diarrheal stools a day, severe abdominal pain, tenesmus and dehydration. Because of these symptoms the nurse should be alert for other problems associated with what disease? A. Chrons disease B.Ulcerative colitis C. Diverticulitis D. Peritonitis Answer: (B) Ulcerative colitis Ulcerative colitis is a chronic inflammatory condition producing edema and ulceration affecting the entire colon.

Ulcerations lead to sloughing that causes stools as many as 10-20 times a day that is filled with blood, pus and mucus. The other symptoms mentioned accompany the problem. 47. A client is being evaluated for cancer of the colon. In preparing the client for barium enema, the nurse should: A.Give laxative the night before and a cleansing enema in the morning before the test B.Render an oil retention enema and give laxative the night before C.Instruct the client to swallow 6 radiopaque tablets the evening before the study D. Place the client on CBR a day before the study Answer: (A) Give laxative the night before and a cleansing enema in the morning before the test Barium enema is the radiologic visualization of the colon using a die. To obtain accurate results in this procedure, the bowels must be emptied of fecal material thus the need for laxative and enema. 48. The client has a good understanding of the means to reduce the chances of colon cancer when he states: A.“I will exercise daily.” B. “I will include more red meat in my diet.” C.“I will have an annual chest x-ray.” D.“I will include more fresh fruits and vegetables in my diet.” Answer: (D) “I will include more fresh fruits and vegetables in my diet.” Numerous aspects of diet and nutrition may contribute to the development of cancer. A low-fiber diet, such as when fresh fruits and vegetables are minimal or lacking in the diet, slows transport of materials through the gut which has been linked to colorectal cancer. 49. Days after abdominal surgery, the client’s wound dehisces. The safest nursing intervention when this occurs is to A. Cover the wound with sterile, moist saline dressing B.Approximate the wound edges with tapes C.Irrigate the wound with sterile saline D.Hold the abdominal contents in place with a sterile gloved hand Answer: (A) Cover the wound with sterile, moist saline dressing Dehiscence is the partial or complete separation of the surgical wound edges. When this occurs, the client is placed in low Fowler’s position and instructed to lie quietly. The wound should be covered to protect it from exposure and the dressing must be sterile to protect it from infection and moist to prevent the dressing from sticking to the wound which can disturb the healing process. 50. An intravenous pyelogram reveals that Paulo, age 35, has a renal calculus. He is believed to have a small stone that will pass spontaneously. To increase the chance of the stone passing, the nurse would instruct the client to force fluids and to A.Strain all urine. B. Ambulate. C. Remain on bed rest. D. Ask for medications to relax him. Answer: (B) Ambulate. Free unattached stones in the urinary tract can be passed out with the urine by ambulation which can mobilize the stone and by increased fluid intake which will flush out the stone during urination. 51. A female client is admitted with a diagnosis of acute renal failure. She is awake, alert, oriented, and complaining of severe back pain, nausea and vomiting and abdominal cramps. Her vital signs are blood pressure 100/70 mm Hg, pulse 110, respirations 30, and oral temperature 100.4°F (38°C). Her electrolytes are sodium 120 mEq/L, potassium 5.2 mEq/L; her urinary output for the first 8 hours is 50 ml. The client is displaying signs of which electrolyte imbalance? A.Hyponatremia B.Hyperkalemia C.Hyperphosphatemia

D.Hypercalcemia Answer: (A) Hyponatremia The normal serum sodium level is 135 – 145 mEq/L. The client’s serum sodium is below normal. Hyponatremia also manifests itself with abdominal cramps and nausea and vomiting 52. Assessing the laboratory findings, which result would the nurse most likely expect to find in a client with chronic renal failure? A.BUN 10 to 30 mg/dl, potassium 4.0 mEq/L, creatinine 0.5 to 1.5 mg/dl B.Decreased serum calcium, blood pH 7.2, potassium 6.5 mEq/L C.BUN 15 mg/dl, increased serum calcium, creatinine l.0 mg/dl D.BUN 35 to 40 mg/dl, potassium 3.5 mEq/L, pH 7.35, decreased serum calcium Answer: (B) Decreased serum calcium, blood pH 7.2, potassium 6.5 mEq/L Chronic renal failure is usually the end result of gradual tissue destruction and loss of renal function. With the loss of renal function, the kidneys ability to regulate fluid and electrolyte and acid base balance results. The serum Ca decreases as the kidneys fail to excrete phosphate, potassium and hydrogen ions are retained. 53. Treatment with hemodialysis is ordered for a client and an external shunt is created. Which nursing action would be of highest priority with regard to the external shunt? A.Heparinize it daily. B.Avoid taking blood pressure measurements or blood samples from the affected arm. C.Change the Silastic tube daily. D.Instruct the client not to use the affected arm. Answer: (B) Avoid taking blood pressure measurements or blood samples from the affected arm. In the client with an external shunt, don’t use the arm with the vascular access site to take blood pressure readings, draw blood, insert IV lines, or give injections because these procedures may rupture the shunt or occlude blood flow causing damage and obstructions in the shunt. 54. Romeo Diaz, age 78, is admitted to the hospital with the diagnosis of benign prostatic hyperplasia (BPH). He is scheduled for a transurethral resection of the prostate (TURP). It would be inappropriate to include which of the following points in the preoperative teaching? A.TURP is the most common operation for BPH. B.Explain the purpose and function of a two-way irrigation system. C.Expect bloody urine, which will clear as healing takes place. D.He will be pain free. Answer: (D) He will be pain free. Surgical interventions involve an experience of pain for the client which can come in varying degrees. Telling the pain that he will be pain free is giving him false reassurance. 55. Roxy is admitted to the hospital with a possible diagnosis of appendicitis. On physical examination, the nurse should be looking for tenderness on palpation at McBurney’s point, which is located in the A.left lower quadrant B.left upper quadrant C.right lower quadrant D.right upper quadrant Answer: (C) right lower quadrant To be exact, the appendix is anatomically located at the Mc Burney’s point at the right iliac area of the right lower quadrant. 56. Mr. Valdez has undergone surgical repair of his inguinal hernia. Discharge teaching should include A.telling him to avoid heavy lifting for 4 to 6 weeks B.instructing him to have a soft bland diet for two weeks C.telling him to resume his previous daily activities without limitations D.recommending him to drink eight glasses of water daily

Answer: (A) telling him to avoid heavy lifting for 4 to 6 weeks The client should avoid lifting heavy objects and any strenuous activity for 4-6 weeks after surgery to prevent stress on the inguinal area. There is no special diet required. The fluid intake of eight glasses a day is good advice but is not a priority in this case. 57. A 30-year-old homemaker fell asleep while smoking a cigarette. She sustained severe burns of the face,neck, anterior chest, and both arms and hands. Using the rule of nines, which is the best estimate of total body-surface area burned? A.18% B.22% C.31% D.40% Answer: (C) 31% Using the Rule of Nine in the estimation of total body surface burned, we allot the following: 9% - head; 9% - each upper extremity; 18%- front chest and abdomen; 18% - entire back; 18% - each lower extremity and 1% - perineum. 58. Nursing care planning is based on the knowledge that the first 24-48 hours post-burn are characterized by: A. An increase in the total volume of intracranial plasma B. Excessive renal perfusion with diuresis C. Fluid shift from interstitial space D. Fluid shift from intravascular space to the interstitial space Answer: (D) Fluid shift from intravascular space to the interstitial space This period is the burn shock stage or the hypovolemic phase. Tissue injury causes vasodilation that results in increase capillary permeability making fluids shift from the intravascular to the interstitial space. This can lead to a decrease in circulating blood volume or hypovolemia which decreases renal perfusion and urine output. 59. If a client has severe bums on the upper torso, which item would be a primary concern? A.Debriding and covering the wounds B. Administering antibiotics C.Frequently observing for hoarseness, stridor, and dyspnea D.Establishing a patent IV line for fluid replacement Answer: (C) Frequently observing for hoarseness, stridor, and dyspnea Burns located in the upper torso, especially resulting from thermal injury related to fires can lead to inhalation burns. This causes swelling of the respiratory mucosa and blistering which can lead to airway obstruction manifested by hoarseness, noisy and difficult breathing. Maintaining a patent airway is a primary concern. 60. Contractures are among the most serious long-term complications of severe burns. If a burn is located on the upper torso, which nursing measure would be least effective to help prevent contractures? A.Changing the location of the bed or the TV set, or both, daily B. Encouraging the client to chew gum and blow up balloons C.Avoiding the use of a pillow for sleep, or placing the head in a position of hyperextension D.Helping the client to rest in the position of maximal comfort Answer: (D) Helping the client to rest in the position of maximal comfort Mobility and placing the burned areas in their functional position can help prevent contracture deformities related to burns. Pain can immobilize a client as he seeks the position where he finds less pain and provides maximal comfort. But this approach can lead to contracture deformities and other complications. 61. An adult is receiving Total Parenteral Nutrition (TPN). Which of the following assessment is essential? A.evaluation of the peripheral IV site B.confirmation that the tube is in the stomach C.assess the bowel sound D.fluid and electrolyte monitoring Answer: (D) fluid and electrolyte monitoring Total parenteral nutrition is a method of providing nutrients to the body by an IV route. The admixture is made up of

proteins, carbohydrates, fats, electrolytes, vitamins, trace minerals and sterile water based on individual client needs. It is intended to improve the clients nutritional status. Because of its composition, it is important to monitor the clients fluid intake and output including electrolytes, blood glucose and weight. 62. Which drug would be least effective in lowering a client's serum potassium level? A.Glucose and insulin B.Polystyrene sulfonate (Kayexalate) C.Calcium glucomite D. Aluminum hydroxide Answer: (D) Aluminum hydroxide Aluminum hydroxide binds dietary phosphorus in the GI tract and helps treat hyperphosphatemia. All the other medications mentioned help treat hyperkalemia and its effects. 63. A nurse is directed to administer a hypotonic intravenous solution. Looking at the following labeled solutions, she should choose A.0.45% NaCl B.0.9% NaCl C. D5W D.D5NSS Answer: (A) 0.45% NaCl Hypotonic solutions like 0.45% NaCl has a lower tonicity that the blood; 0.9% NaCl and D5W are isotonic solutions with same tonicity as the blood; and D5NSS is hypertonic with a higher tonicity thab the blood. 64. A patient is hemorrhaging from multiple trauma sites. The nurse expects that compensatory mechanisms associated with hypovolemia would cause all of the following symptoms EXCEPT A.hypertension B.oliguria C.tachycardia D.tachypnea Answer: (A) hypertension In hypovolemia, one of the compenasatory mechanisms is activation of the sympathetic nervous system that increases the RR & PR and helps restore the BP to maintain tissue perfusion but not cause a hypertension. The SNS stimulation constricts renal arterioles that increases release of aldosterone, decreases glomerular filtration and increases sodium & water reabsorption that leads to oliguria. 65. Maria Sison, 40 years old, single, was admitted to the hospital with a diagnosis of Breast Cancer. She was scheduled for radical mastectomy. Nursing care during the preoperative period should consist of A.assuring Maria that she will be cured of cancer B.assessing Maria's expectations and doubts C.maintaining a cheerful and optimistic environment D.keeping Maria's visitors to a minimum so she can have time for herself Answer: (B) assessing Maria's expectations and doubts Assessing the client’s expectations and doubts will help lessen her fears and anxieties. The nurse needs to encourage the client to verbalize and to listen and correctly provide explanations when needed. 66. Maria refuses to acknowledge that her breast was removed. She believes that her breast is intact under the dressing. The nurse should A.call the MD to change the dressing so Kathy can see the incision B.recognize that Kathy is experiencing denial, a normal stage of the grieving process C.reinforce Kathy’s belief for several days until her body can adjust to stress of surgery. D.remind Kathy that she needs to accept her diagnosis so that she can begin rehabilitation exercises. Answer: (B) recognize that Kathy is experiencing denial, a normal stage of the grieving process A person grieves to a loss of a significant object. The initial stage in the grieving process is denial, then anger, followed by bargaining, depression and last acceptance. The nurse should show acceptance of the patient’s feelings and encourage verbalization.

67. A chemotherapeutic agent 5FU is ordered as an adjunct measure to surgery. Which of the ff. statements about chemotherapy is true? A.it is a local treatment affecting only tumor cells B.it affects both normal and tumor cells C.it has been proven as a complete cure for cancer D.it is often used as a palliative measure. Answer: (B) it affects both normal and tumor cells Chemotherapeutic agents are given to destroy the actively proliferating cancer cells. But these agents cannot differentiate the abnormal actively proliferating cancer cells from those that are actively proliferating normal cells like the cells of the bone marrow, thus the effect of bone marrow depression. 68. Which is an incorrect statement pertaining to the following procedures for cancer diagnostics? A.Biopsy is the removal of suspicious tissue and the only definitive method to diagnose cancer B. Ultrasonography detects tissue density changes difficult to observe by X-ray via sound waves. C.CT scanning uses magnetic fields and radio frequencies to provide cross-sectional view of tumor D.Endoscopy provides direct view of a body cavity to detect abnormality. Answer: (C) CT scanning uses magnetic fields and radio frequencies to provide cross-sectional view of tumor CT scan uses narrow beam x-ray to provide cross-sectional view. MRI uses magnetic fields and radio frequencies to detect tumors. 69. A post-operative complication of mastectomy is lymphedema. This can be prevented by A.ensuring patency of wound drainage tube B.placing the arm on the affected side in a dependent position C.restricting movement of the affected arm D.frequently elevating the arm of the affected side above the level of the heart. Answer: (D) frequently elevating the arm of the affected side above the level of the heart. Elevating the arm above the level of the heart promotes good venous return to the heart and good lymphatic drainage thus preventing swelling. 70. Which statement by the client indicates to the nurse that the patient understands precautions necessary during internal radiation therapy for cancer of the cervix? A. “I should get out of bed and walk around in my room.” B. “My 7 year old twins should not come to visit me while I’m receiving treatment.” C. “I will try not to cough, because the force might make me expel the application.” D. “I know that my primary nurse has to wear one of those badges like the people in the x-ray department, but they are not necessary for anyone else who comes in here.” Answer: (B) “My 7 year old twins should not come to visit me while I’m receiving treatment.” Children have cells that are normally actively dividing in the process of growth. Radiation acts not only against the abnormally actively dividing cells of cancer but also on the normally dividing cells thus affecting the growth and development of the child and even causing cancer itself. 71. High uric acid levels may develop in clients who are receiving chemotherapy. This is caused by: A.The inability of the kidneys to excrete the drug metabolites B. Rapid cell catabolism C.Toxic effect of the antibiotic that are given concurrently D. The altered blood ph from the acid medium of the drugs Answer: (B) Rapid cell catabolism One of the oncologic emergencies, the tumor lysis syndrome, is caused by the rapid destruction of large number of tumor cells. . Intracellular contents are released, including potassium and purines, into the bloodstream faster than the body can eliminate them. The purines are converted in the liver to uric acid and released into the blood causing hyperuricemia. They can precipitate in the kidneys and block the tubules causing acute renal failure.

72. Which of the following interventions would be included in the care of plan in a client with cervical implant? A.Frequent ambulation B.Unlimited visitors C.Low residue diet D.Vaginal irrigation every shift Answer: (C) Low residue diet It is important for the nurse to remember that the implant be kept intact in the cervix during therapy. Mobility and vaginal irrigations are not done. A low residue diet will prevent bowel movement that could lead to dislodgement of the implant. Patient is also strictly isolated to protect other people from the radiation emissions 73. Which nursing measure would avoid constriction on the affected arm immediately after mastectomy? A.Avoid BP measurement and constricting clothing on the affected arm B. Active range of motion exercises of the arms once a day. C.Discourage feeding, washing or combing with the affected arm D.Place the affected arm in a dependent position, below the level of the heart Answer: (A) Avoid BP measurement and constricting clothing on the affected arm A BP cuff constricts the blood vessels where it is applied. BP measurements should be done on the unaffected arm to ensure adequate circulation and venous and lymph drainage in the affected arm 74. A client suffering from acute renal failure has an unexpected increase in urinary output to 150ml/hr. The nurse assesses that the client has entered the second phase of acute renal failure. Nursing actions throughout this phase include observation for signs and symptoms of A.Hypervolemia, hypokalemia, and hypernatremia. B.Hypervolemia, hyperkalemia, and hypernatremia. C.Hypovolemia, wide fluctuations in serum sodium and potassium levels. D.Hypovolemia, no fluctuation in serum sodium and potassium levels. Answer: (C) Hypovolemia, wide fluctuations in serum sodium and potassium levels. The second phase of ARF is the diuretic phase or high output phase. The diuresis can result in an output of up to 10L/day of dilute urine. Loss of fluids and electrolytes occur. 75. An adult has just been brought in by ambulance after a motor vehicle accident. When assessing the client, the nurse would expect which of the following manifestations could have resulted from sympathetic nervous system stimulation? A.A rapid pulse and increased RR B. Decreased physiologic functioning C.Rigid posture and altered perceptual focus D.Increased awareness and attention Answer: (A) A rapid pulse and increased RR The fight or flight reaction of the sympathetic nervous system occurs during stress like in a motor vehicular accident. This is manifested by increased in cardiovascular function and RR to provide the immediate needs of the body for survival. 76. Ms. Sy undergoes surgery and the abdominal aortic aneurysm is resected and replaced with a graft. When she arrives in the RR she is still in shock. The nurse's priority should be A.placing her in a trendeleburg position B.putting several warm blankets on her C.monitoring her hourly urine output D.assessing her VS especially her RR Answer: (D) assessing her VS especially her RR Shock is characterized by reduced tissue and organ perfusion and eventual organ dysfunction and failure. Checking

on the VS especially the RR, which detects need for oxygenation, is a priority to help detect its progress and provide for prompt management before the occurrence of complications. 77. A major goal for the client during the first 48 hours after a severe bum is to prevent hypovolemic shock. The best indicator of adequate fluid balance during this period is A.Elevated hematocrit levels. B.Urine output of 30 to 50 ml/hr. C.Change in level of consciousness. D.Estimate of fluid loss through the burn eschar. Answer: (B) Urine output of 30 to 50 ml/hr. Hypovolemia is a decreased in circulatory volume. This causes a decrease in tissue perfusion to the different organs of the body. Measuring the hourly urine output is the most quantifiable way of measuring tissue perfusion to the organs. Normal renal perfusion should produce 1ml/kg of BW/min. An output of 30-50 ml/hr is considered adequate and indicates good fluid balance. 78. A thoracentesis is performed on a chest-injured client, and no fluid or air is found. Blood and fluids is administered intravenously (IV), but the client's vital signs do not improve. A central venous pressure line is inserted, and the initial reading is 20 cm H^O. The most likely cause of these findings is which of the following? A.Spontaneous pneumothorax B.Ruptured diaphragm C.Hemothorax D.Pericardial tamponade Answer: (D) Pericardial tamponade Pericardial tamponade occurs when there is presence of fluid accumulation in the pericardial space that compresses on the ventricles causing a decrease in ventricular filling and stretching during diastole with a decrease in cardiac output. . This leads to right atrial and venous congestion manifested by a CVP reading above normal. 79. Intervention for a pt. who has swallowed a Muriatic Acid includes all of the following except A.administering an irritant that will stimulate vomiting B.aspirating secretions from the pharynx if respirations are affected C.neutralizing the chemical D.washing the esophagus with large volumes of water via gastric lavage Answer: (A) administering an irritant that will stimulate vomiting Swallowing of corrosive substances causes severe irritation and tissue destruction of the mucous membrane of the GI tract. Measures are taken to immediately remove the toxin or reduce its absorption. For corrosive poison ingestion, such as in muriatic acid where burn or perforation of the mucosa may occur, gastric emptying procedure is immediately instituted, This includes gastric lavage and the administration of activated charcoal to absorb the poison. Administering an irritant with the concomitant vomiting to remove the swallowed poison will further cause irritation and damage to the mucosal lining of the digestive tract. Vomiting is only indicated when non-corrosive poison is swallowed. 80. Which initial nursing assessment finding would best indicate that a client has been successfully resuscitated after a cardio-respiratory arrest? A.Skin warm and dry B.Pupils equal and react to light C.Palpable carotid pulse D. Positive Babinski's reflex Answer: (C) Palpable carotid pulse Presence of a palpable carotid pulse indicates the return of cardiac function which, together with the return of breathing, is the primary goal of CPR. Pulsations in arteries indicates blood flowing in the blood vessels with each cardiac contraction. Signs of effective tissue perfusion will be noted after. 81. Chemical burn of the eye are treated with A.local anesthetics and antibacterial drops for 24 – 36 hrs. B.hot compresses applied at 15-minute intervals C.Flushing of the lids, conjunctiva and cornea with tap or preferably sterile water

D.cleansing the conjunctiva with a small cotton-tipped applicator Answer: (C) Flushing of the lids, conjunctiva and cornea with tap or preferably sterile water Prompt treatment of ocular chemical burns is important to prevent further damage. Immediate tap-water eye irrigation should be started on site even before transporting the patient to the nearest hospital facility. In the hospital, copious irrigation with normal saline, instillation of local anesthetic and antibiotic is done. 82. The Heimlich maneuver (abdominal thrust), for acute airway obstruction, attempts to: A.Force air out of the lungs B. Increase systemic circulation C.Induce emptying of the stomach D. Put pressure on the apex of the heart Answer: (A) Force air out of the lungs The Heimlich maneuver is used to assist a person choking on a foreign object. The pressure from the thrusts lifts the diaphragm, forces air out of the lungs and creates an artificial cough that expels the aspirated material. 83. John, 16 years old, is brought to the ER after a vehicular accident. He is pronounced dead on arrival. When his parents arrive at the hospital, the nurse should: A.ask them to stay in the waiting area until she can spend time alone with them B. speak to both parents together and encourage them to support each other and express their emotions freely C.Speak to one parent at a time so that each can ventilate feelings of loss without upsetting the other D.ask the MD to medicate the parents so they can stay calm to deal with their son’s death. Answer: (B) speak to both parents together and encourage them to support each other and express their emotions freely Sudden death of a family member creates a state of shock on the family. They go into a stage of denial and anger in their grieving. Assisting them with information they need to know, answering their questions and listening to them will provide the needed support for them to move on and be of support to one another. 84. An emergency treatment for an acute asthmatic attack is Adrenaline 1:1000 given hypodermically. This is given to: A.increase BP B.decrease mucosal swelling C. relax the bronchial smooth muscle D.decrease bronchial secretions Answer: (C) relax the bronchial smooth muscle Acute asthmatic attack is characterized by severe bronchospasm which can be relieved by the immediate administration of bronchodilators. Adrenaline or Epinephrine is an adrenergic agent that causes bronchial dilation by relaxing the bronchial smooth muscles. 85. A nurse is performing CPR on an adult patient. When performing chest compressions, the nurse understands the correct hand placement is located over the A.upper half of the sternum B.upper third of the sternum C.lower half of the sternum D.lower third of the sternum Answer: (C) lower half of the sternum The exact and safe location to do cardiac compression is the lower half of the sternum. Doing it at the lower third of the sternum may cause gastric compression which can lead to a possible aspiration. 86. The nurse is performing an eye examination on an elderly client. The client states ‘My vision is blurred, and I don’t easily see clearly when I get into a dark room.” The nurse best response is: A. “You should be grateful you are not blind.” B. “As one ages, visual changes are noted as part of degenerative changes. This is normal.” C. “You should rest your eyes frequently.” D. “You maybe able to improve you vision if you move slowly.”

Answer: (B) “As one ages, visual changes are noted as part of degenerative changes. This is normal.” Aging causes less elasticity of the lens affecting accommodation leading to blurred vision. The muscles of the iris increase in stiffness and the pupils dilate slowly and less completely so that it takes the older person to adjust when going to and from light and dark environment and needs brighter light for close vision. 87. Which of the following activities is not encouraged in a patient after an eye surgery? A.sneezing, coughing and blowing the nose B.straining to have a bowel movement C.wearing tight shirt collars D.sexual intercourse Answer: (D) sexual intercourse To reduce increases in IOP, teach the client and family about activity restrictions. Sexual intercourse can cause a sudden rise in IOP. 88. Which of the following indicates poor practice in communicating with a hearing-impaired client? A.Use appropriate hand motions B.Keep hands and other objects away from your mouth when talking to the client C.Speak clearly in a loud voice or shout to be heard D.Converse in a quiet room with minimal distractions Answer: (C) Speak clearly in a loud voice or shout to be heard Shouting raises the frequency of the sound and often makes understanding the spoken words difficult. It is enough for the nurse to speak clearly and slowly. 89. A client is to undergo lumbar puncture. Which is least important information about LP? A.Specimens obtained should be labeled in their proper sequence. B.It may be used to inject air, dye or drugs into the spinal canal. C.Assess movements and sensation in the lower extremities after the D.Force fluids before and after the procedure. Answer: (D) Force fluids before and after the procedure. LP involves the removal of some amount of spinal fluid. To facilitate CSF production, the client is instructed to increase fluid intake to 3L, unless contraindicated, for 24 to 48 hrs after the procedure. 90. A client diagnosed with cerebral thrombosis is scheduled for cerebral angiography. Nursing care of the client includes the following EXCEPT A.Inform the client that a warm, flushed feeling and a salty taste may be B.Maintain pressure dressing over the site of puncture and check for C.Check pulse, color and temperature of the extremity distal to the site of D.Kept the extremity used as puncture site flexed to prevent bleeding. Answer: (D) Kept the extremity used as puncture site flexed to prevent bleeding. Angiography involves the threading of a catheter through an artery which can cause trauma to the endothelial lining of the blood vessel. The platelets are attracted to the area causing thrombi formation. This is further enhanced by the slowing of blood flow caused by flexion of the affected extremity. The affected extremity must be kept straight and immobilized during the duration of the bedrest after the procedure. Ice bag can be applied intermittently to the puncture site. 91. Which is considered as the earliest sign of increased ICP that the nurse should closely observed for? A.abnormal respiratory pattern B.rising systolic and widening pulse pressure C.contralateral hemiparesis and ipsilateral dilation of the pupils D.progression from restlessness to confusion and disorientation to lethargy Answer: (D) progression from restlessness to confusion and disorientation to lethargy The first major effect of increasing ICP is a decrease in cerebral perfusion causing hypoxia that produces a progressive alteration in the LOC. This is initially manifested by restlessness.

92. Which is irrelevant in the pharmacologic management of a client with CVA? A.Osmotic diuretics and corticosteroids are given to decrease cerebral edema B.Anticonvulsants are given to prevent seizures C.Thrombolytics are most useful within three hours of an occlusive CVA D.Aspirin is used in the acute management of a completed stroke. Answer: (D) Aspirin is used in the acute management of a completed stroke. The primary goal in the management of CVA is to improve cerebral tissue perfusion. Aspirin is a platelet deaggregator used in the prevention of recurrent or embolic stroke but is not used in the acute management of a completed stroke as it may lead to bleeding. 93. What would be the MOST therapeutic nursing action when a client’s expressive aphasia is severe? A.Anticipate the client wishes so she will not need to talk B.Communicate by means of questions that can be answered by the client shaking the head C. Keep us a steady flow rank to minimize silence D.Encourage the client to speak at every possible opportunity. Answer: (D) Encourage the client to speak at every possible opportunity. Expressive or motor aphasia is a result of damage in the Broca’s area of the frontal lobe. It is amotor speech problem in which the client generally understands what is said but is unable to communicate verbally. The patient can best he helped therefore by encouraging him to communicate and reinforce this behavior positively. 94. A client with head injury is confused, drowsy and has unequal pupils. Which of the following nursing diagnosis is most important at this time? A.altered level of cognitive function B.high risk for injury C.altered cerebral tissue perfusion D.sensory perceptual alteration Answer: (C) altered cerebral tissue perfusion The observations made by the nurse clearly indicate a problem of decrease cerebral perfusion. Restoring cerebral perfusion is most important to maintain cerebral functioning and prevent further brain damage. 95. Which nursing diagnosis is of the highest priority when caring for a client with myasthenia gravis? A.Pain B. High risk for injury related to muscle weakness C.Ineffective coping related to illness D.Ineffective airway clearance related to muscle weakness Answer: (D) Ineffective airway clearance related to muscle weakness Myasthenia gravis causes a failure in the transmission of nerve impulses at the neuromuscular junction which may be due to a weakening or decrease in acetylcholine receptor sites. This leads to sporadic, progressive weakness or abnormal fatigability of striated muscles that eventually causes loss of function. The respiratory muscles can become weak with decreased tidal volume and vital capacity making breathing and clearing the airway through coughing difficult. The respiratory muscle weakness may be severe enough to require and emergency airway and mechanical ventilation. 96. The client has clear drainage from the nose and ears after a head injury. How can the nurse determine if the drainage is CSF? A. Measure the ph of the fluid B. Measure the specific gravity of the fluid C.Test for glucose D.Test for chlorides Answer: (C) Test for glucose The CSF contains a large amount of glucose which can be detected by using glucostix. A positive result with the drainage indicate CSF leakage.

97. The nurse includes the important measures for stump care in the teaching plan for a client with an amputation. Which measure would be excluded from the teaching plan? A.Wash, dry, and inspect the stump daily. B.Treat superficial abrasions and blisters promptly. C.Apply a "shrinker" bandage with tighter arms around the proximal end of the affected limb. D.Toughen the stump by pushing it against a progressively harder substance (e.g., pillow on a foot-stool). Answer: (C) Apply a "shrinker" bandage with tighter arms around the proximal end of the affected limb. The “shrinker” bandage is applied to prevent swelling of the stump. It should be applied with the distal end with the tighter arms. Applying the tighter arms at the proximal end will impair circulation and cause swelling by reducing venous flow. 98. A 70-year-old female comes to the clinic for a routine checkup. She is 5 feet 4 inches tall and weighs 180 pounds. Her major complaint is pain in her joints. She is retired and has had to give up her volunteer work because of her discomfort. She was told her diagnosis was osteoarthritis about 5 years ago. Which would be excluded from the clinical pathway for this client? A.Decrease the calorie count of her daily diet. B.Take warm baths when arising. C.Slide items across the floor rather than lift them. D.Place items so that it is necessary to bend or stretch to reach them. Answer: (D) Place items so that it is necessary to bend or stretch to reach them. Patients with osteoarthritis have decreased mobility caused by joint pain. Over-reaching and stretching to get an object are to be avoided as this can cause more pain and can even lead to falls. The nurse should see to it therefore that objects are within easy reach of the patient. 99. A client is admitted from the emergency department with severe-pain and edema in the right foot. His diagnosis is gouty arthritis. When developing a plan of care, which action would have the highest priority? A.Apply hot compresses to the affected joints. B.Stress the importance of maintaining good posture to prevent deformities. C.Administer salicylates to minimize the inflammatory reaction. D.Ensure an intake of at least 3000 ml of fluid per day. Answer: (D) Ensure an intake of at least 3000 ml of fluid per day. Gouty arthritis is a metabolic disease marked by urate deposits that cause painful arthritic joints. The patient should be urged to increase his fluid intake to prevent the development of urinary uric acid stones. 100. A client had a laminectomy and spinal fusion yesterday. Which statement is to be excluded from your plan of care? A.Before log rolling, place a pillow under the client's head and a pillow between the client's legs. B.Before log rolling, remove the pillow from under the client's head and use no pillows between the client's legs. C.Keep the knees slightly flexed while the client is lying in a semi-Fowler's position in bed. D.Keep a pillow under the client's head as needed for comfort. Answer: (B) Before log rolling, remove the pillow from under the client's head and use no pillows between the client's legs. Following a laminectomy and spinal fusion, it is important that the back of the patient be maintained in straight alignment and to support the entire vertebral column to promote complete healing. 101. The nurse is assisting in planning care for a client with a diagnosis of immune deficiency. The nurse would incorporate which of the ff. as a priority in the plan of care? A.providing emotional support to decrease fear B.protecting the client from infection C.encouraging discussion about lifestyle changes D.identifying factors that decreased the immune function Answer: (B) protecting the client from infection

Immunodeficiency is an absent or depressed immune response that increases susceptibility to infection. So it is the nurse’s primary responsibility to protect the patient from infection. 102. Joy, an obese 32 year old, is admitted to the hospital after an automobile accident. She has a fractured hip and is brought to the OR for surgery. After surgery Joy is to receive a piggy-back of Clindamycin phosphate (Cleocin) 300 mg in 50 ml of D5W. The piggyback is to infuse in 20 minutes. The drop factor of the IV set is 10 gtt/ml. The nurse should set the piggyback to flow at: A.25 gtt/min B.30 gtt/min C.35 gtt/min D.45 gtt/min Answer: (A) 25 gtt/min To get the correct flow rate: multiply the amount to be infused (50 ml) by the drop factor (10) and divide the result by the amount of time in minutes (20) 103. The day after her surgery Joy asks the nurse how she might lose weight. Before answering her question, the nurse should bear in mind that long-term weight loss best occurs when: A.Fats are controlled in the diet B.Eating habits are altered C.Carbohydrates are regulated D.Exercise is part of the program Answer: (B) Eating habits are altered For weight reduction to occur and be maintained, a new dietary program, with a balance of foods from the basic four food groups, must be established and continued 104. The nurse teaches Joy, an obese client, the value of aerobic exercises in her weight reduction program. The nurse would know that this teaching was effective when Joy says that exercise will: A.Increase her lean body mass B.Lower her metabolic rate C.Decrease her appetite D.Raise her heart rate Answer: (A) Increase her lean body mass Increased exercise builds skeletal muscle mass and reduces excess fatty tissue. 105. The physician orders non-weight bearing with crutches for Joy, who had surgery for a fractured hip. The most important activity to facilitate walking with crutches before ambulation begun is: A.Exercising the triceps, finger flexors, and elbow extensors B.Sitting up at the edge of the bed to help strengthen back muscles C.Doing isometric exercises on the unaffected leg D.Using the trapeze frequently for pull-ups to strengthen the biceps muscles Answer: (A) Exercising the triceps, finger flexors, and elbow extensors These sets of muscles are used when walking with crutches and therefore need strengthening prior to ambulation. 106. The nurse recognizes that a client understood the demonstration of crutch walking when she places her weight on: A.The palms and axillary regions B.Both feet placed wide apart C.The palms of her hands D.Her axillary regions

Answer: (C) The palms of her hands The palms should bear the client’s weight to avoid damage to the nerves in the axilla (brachial plexus) 107. Joey is a 46 year-old radio technician who is admitted because of mild chest pain. He is 5 feet, 8 inches tall and weighs 190 pounds. He is diagnosed with a myocardial infarct. Morphine sulfate, Diazepam (Valium) and Lidocaine are prescribed. The physician orders 8 mg of Morphine Sulfate to be given IV. The vial on hand is labeled 1 ml/ 10 mg. The nurse should administer: A.8 minims B.10 minims C.12 minims D.15 minims Answer: (C) 12 minims Using ratio and proportion 8 mg/10 mg = X minims/15 minims 10 X= 120 X = 12 minims The nurse will administer 12 minims intravenously equivalent to 8mg Morphine Sulfate 108. Joey asks the nurse why he is receiving the injection of Morphine after he was hospitalized for severe anginal pain. The nurse replies that it: A.Will help prevent erratic heart beats B.Relieves pain and decreases level of anxiety C.Decreases anxiety D.Dilates coronary blood vessels Answer: (B) Relieves pain and decreases level of anxiety Morphine is a specific central nervous system depressant used to relieve the pain associated with myocardial infarction. It also decreases anxiety and apprehension and prevents cardiogenic shock by decreasing myocardial oxygen demand. 109. Oxygen 3L/min by nasal cannula is prescribed for Joey who is admitted to the hospital for chest pain. The nurse institutes safety precautions in the room because oxygen: A.Converts to an alternate form of matter B.Has unstable properties C.Supports combustion D.Is flammable Answer: (C) Supports combustion The nurse should know that Oxygen is necessary to produce fire, thus precautionary measures are important regarding its use. 110. Myra is ordered laboratory tests after she is admitted to the hospital for angina. The isoenzyme test that is the most reliable early indicator of myocardial insult is: A.SGPT B.LDH C.CK-MB D.AST Answer: (C) CK-MB The cardiac marker, Creatinine phosphokinase (CPK) isoenzyme levels, especially the MB sub-unit which is cardiospecific, begin to rise in 3-6 hours, peak in 12-18 hours and are elevated 48 hours after the occurrence of the infarct. They are therefore most reliable in assisting with early diagnosis. The cardiac markers elevate as a result of myocardial tissue damage. 111. An early finding in the EKG of a client with an infarcted mycardium would be:

A.Disappearance of Q waves B.Elevated ST segments C.Absence of P wave D.Flattened T waves Answer: (B) Elevated ST segments This is a typical early finding after a myocardial infarct because of the altered contractility of the heart. The other choices are not typical of MI. 112. Jose, who had a myocardial infarction 2 days earlier, has been complaining to the nurse about issues related to his hospital stay. The best initial nursing response would be to: A.Allow him to release his feelings and then leave him alone to allow him to regain his composure B.Refocus the conversation on his fears, frustrations and anger about his condition C.Explain how his being upset dangerously disturbs his need for rest D.Attempt to explain the purpose of different hospital routines Answer: (B) Refocus the conversation on his fears, frustrations and anger about his condition This provides the opportunity for the client to verbalize feelings underlying behavior and helpful in relieving anxiety. Anxiety can be a stressor which can activate the sympathoadrenal response causing the release of catecholamines that can increase cardiac contractility and workload that can further increase myocardial oxygen demand. 113. Twenty four hours after admission for an Acute MI, Jose’s temperature is noted at 39.3 C. The nurse monitors him for other adaptations related to the pyrexia, including: A.Shortness of breath B.Chest pain C.Elevated blood pressure D.Increased pulse rate Answer: (D) Increased pulse rate Fever causes an increase in the body’s metabolism, which results in an increase in oxygen consumption and demand. This need for oxygen increases the heart rate, which is reflected in the increased pulse rate. Increased BP, chest pain and shortness of breath are not typically noted in fever. 114. Jose, who is admitted to the hospital for chest pain, asks the nurse, “Is it still possible for me to have another heart attack if I watch my diet religiously and avoid stress?” The most appropriate initial response would be for the nurse to: A.Suggest he discuss his feelings of vulnerability with his physician. B.Tell him that he certainly needs to be especially careful about his diet and lifestyle. C.Avoid giving him direct information and help him explore his feelings D.Recognize that he is frightened and suggest he talk with the psychiatrist or counselor. Answer: (C) Avoid giving him direct information and help him explore his feelings To help the patient verbalize and explore his feelings, the nurse must reflect and analyze the feelings that are implied in the client’s question. The focus should be on collecting data to minister to the client’s psychosocial needs. 115. Ana, 55 years old, is admitted to the hospital to rule out pernicious anemia. A Schilling test is ordered for Ana. The nurse recognizes that the primary purpose of the Schilling test is to determine the client’s ability to: A.Store vitamin B12 B.Digest vitamin B12 C.Absorb vitamin B12 D.Produce vitamin B12 Answer: (C) Absorb vitamin B12 Pernicious anemia is caused by the inability to absorb vitamin B12 in the stomach due to a lack of intrinsic factor in the gastric juices. In the Schilling test, radioactive vitamin B12 is administered and its absorption and excretion can be ascertained through the urine.

116. Ana is diagnosed to have Pernicious anemia. The physician orders 0.2 mg of Cyanocobalamin (Vitamin B12) IM. Available is a vial of the drug labeled 1 ml= 100 mcg. The nurse should administer: A.0.5 ml B.1.0 ml C.1.5 ml D.2.0 ml Answer: (D) 2.0 ml First convert milligrams to micrograms and then use ratio and proportion (0.2 mg= 200 mcg) 200 mcg : 100 mcg= X ml : ml 100 X= 200 X = 2 ml. Inject 2 ml. to give 0.2 mg of Cyanocobalamin. 117. Health teachings to be given to a client with Pernicious Anemia regarding her therapeutic regimen concerning Vit. B12 will include: A.Oral tablets of Vitamin B12 will control her symptoms B.IM injections are required for daily control C.IM injections once a month will maintain control D.Weekly Z-track injections provide needed control Answer: (C) IM injections once a month will maintain control Deep IM injections bypass B12 absorption defect in the stomach due to lack of intrinsic factor, the transport carrier component of gastric juices. A monthly dose is usually sufficient since it is stored in active body tissues such as the liver, kidney, heart, muscles, blood and bone marrow 118. The nurse knows that a client with Pernicious Anemia understands the teaching regarding the vitamin B12 injections when she states that she must take it: A.When she feels fatigued B.During exacerbations of anemia C.Until her symptoms subside D.For the rest of her life Answer: (D) For the rest of her life Since the intrinsic factor does not return to gastric secretions even with therapy, B12 injections will be required for the remainder of the client’s life. 119. Arthur Cruz, a 45 year old artist, has recently had an abdominoperineal resection and colostomy. Mr. Cruz accuses the nurse of being uncomfortable during a dressing change, because his “wound looks terrible.” The nurse recognizes that the client is using the defense mechanism known as: A.Reaction Formation B.Sublimation C.Intellectualization D.Projection Answer: (D) Projection Projection is the attribution of unacceptable feelings and emotions to others which may indicate the patients nonacceptance of his condition. 120. When preparing to teach a client with colostomy how to irrigate his colostomy, the nurse should plan to perform the procedure: A.When the client would have normally had a bowel movement B.After the client accepts he had a bowel movement C.Before breakfast and morning care D.At least 2 hours before visitors arrive Answer: (A) When the client would have normally had a bowel movement Irrigation should be performed at the time the client normally defecated before the colostomy to maintain continuity in lifestyle and usual bowel function/habit.

121. When observing an ostomate do a return demonstration of the colostomy irrigation, the nurse notes that he needs more teaching if he: A.Stops the flow of fluid when he feels uncomfortable B.Lubricates the tip of the catheter before inserting it into the stoma C.Hangs the bag on a clothes hook on the bathroom door during fluid insertion D.Discontinues the insertion of fluid after only 500 ml of fluid has been instilled Answer: (C) Hangs the bag on a clothes hook on the bathroom door during fluid insertion The irrigation bag should be hung 12-18 inches above the level of the stoma; a clothes hook is too high which can create increase pressure and sudden intestinal distention and cause abdominal discomfort to the patient. 122. When doing colostomy irrigation at home, a client with colostomy should be instructed to report to his physician : A.Abdominal cramps during fluid inflow B.Difficulty in inserting the irrigating tube C.Passage of flatus during expulsion of feces D.Inability to complete the procedure in half an hour Answer: (B) Difficulty in inserting the irrigating tube Difficulty of inserting the irrigating tube indicates stenosis of the stoma and should be reported to the physician. Abdominal cramps and passage of flatus can be expected during colostomy irrigations. The procedure may take longer than half an hour. 123. A client with colostomy refuses to allow his wife to see the incision or stoma and ignores most of his dietary instructions. The nurse on assessing this data, can assume that the client is experiencing: A.A reaction formation to his recent altered body image. B.A difficult time accepting reality and is in a state of denial. C.Impotency due to the surgery and needs sexual counseling D.Suicide thoughts and should be seen by psychiatrist Answer: (B) A difficult time accepting reality and is in a state of denial. As long as no one else confirms the presence of the stoma and the client does not need to adhere to a prescribed regimen, the client’s denial is supported 124. The nurse would know that dietary teaching had been effective for a client with colostomy when he states that he will eat: A.Food low in fiber so that there is less stool B.Everything he ate before the operation but will avoid those foods that cause gas C.Bland foods so that his intestines do not become irritated D.Soft foods that are more easily digested and absorbed by the large intestines Answer: (B) Everything he ate before the operation but will avoid those foods that cause gas There is no special diets for clients with colostomy. These clients can eat a regular diet. Only gas-forming foods that cause distention and discomfort should be avoided. 125. Eddie, 40 years old, is brought to the emergency room after the crash of his private plane. He has suffered multiple crushing wounds of the chest, abdomen and legs. It is feared his leg may have to be amputated. When Eddie arrives in the emergency room, the assessment that assume the greatest priority are: A.Level of consciousness and pupil size B.Abdominal contusions and other wounds C.Pain, Respiratory rate and blood pressure D.Quality of respirations and presence of pulsesQuality of respirations and presence of pulses Answer: (D) Quality of respirations and presence of pulsesQuality of respirations and presence of pulses Respiratory and cardiovascular functions are essential for oxygenation. These are top priorities to trauma management. Basic life functions must be maintained or reestablished

126. Eddie, a plane crash victim, undergoes endotracheal intubation and positive pressure ventilation. The most immediate nursing intervention for him at this time would be to: A.Facilitate his verbal communication B.Maintain sterility of the ventilation system C.Assess his response to the equipment D.Prepare him for emergency surgery Answer: (C) Assess his response to the equipment It is a primary nursing responsibility to evaluate effect of interventions done to the client. Nothing is achieved if the equipment is working and the client is not responding 127. A chest tube with water seal drainage is inserted to a client following a multiple chest injury. A few hours later, the client’s chest tube seems to be obstructed. The most appropriate nursing action would be to A.Prepare for chest tube removal B.Milk the tube toward the collection container as ordered C.Arrange for a stat Chest x-ray film. D.Clam the tube immediately Answer: (B) Milk the tube toward the collection container as ordered This assists in moving blood, fluid or air, which may be obstructing drainage, toward the collection chamber 128. The observation that indicates a desired response to thoracostomy drainage of a client with chest injury is: A.Increased breath sounds B.Constant bubbling in the drainage chamber C.Crepitus detected on palpation of chest D.Increased respiratory rate Answer: (A) Increased breath sounds The chest tube normalizes intrathoracic pressure and restores negative intra-pleural pressure, drains fluid and air from the pleural space, and improves pulmonary function 129. In the evaluation of a client’s response to fluid replacement therapy, the observation that indicates adequate tissue perfusion to vital organs is: A.Urinary output is 30 ml in an hour B.Central venous pressure reading of 2 cm H2O C.Pulse rates of 120 and 110 in a 15 minute period D.Blood pressure readings of 50/30 and 70/40 within 30 minutes Answer: (A) Urinary output is 30 ml in an hour A rate of 30 ml/hr is considered adequate for perfusion of kidney, heart and brain. 130. A client with multiple injury following a vehicular accident is transferred to the critical care unit. He begins to complain of increased abdominal pain in the left upper quadrant. A ruptured spleen is diagnosed and he is scheduled for emergency splenectomy. In preparing the client for surgery, the nurse should emphasize in his teaching plan the: A.Complete safety of the procedure B.Expectation of postoperative bleeding C.Risk of the procedure with his other injuries D.Presence of abdominal drains for several days after surgery Answer: (D) Presence of abdominal drains for several days after surgery Drains are usually inserted into the splenic bed to facilitate removal of fluid in the area that could lead to abscess formation. 131. To promote continued improvement in the respiratory status of a client following chest tube removal after a chest surgery for multiple rib fracture, the nurse should: A.Encourage bed rest with active and passive range of motion exercises

B.Encourage frequent coughing and deep breathing C.Turn him from side to side at least every 2 hours D.Continue observing for dyspnea and crepitus Answer: (B) Encourage frequent coughing and deep breathing This nursing action prevents atelectasis and collection of respiratory secretions and promotes adequate ventilation and gas exchange. 132. A client undergoes below the knee amputation following a vehicular accident. Three days postoperatively, the client is refusing to eat, talk or perform any rehabilitative activities. The best initial nursing approach would be to: A.Give him explanations of why there is a need to quickly increase his activity B.Emphasize repeatedly that with as prosthesis, he will be able to return to his normal lifestyle C.Appear cheerful and non-critical regardless of his response to attempts at intervention D.Accept and acknowledge that his withdrawal is an initially normal and necessary part of grieving Answer: (D) Accept and acknowledge that his withdrawal is an initially normal and necessary part of grieving The withdrawal provides time for the client to assimilate what has occurred and integrate the change in the body image. Acceptance of the client’s behavior is an important factor in the nurse’s intervention. 133. The key factor in accurately assessing how body image changes will be dealt with by the client is the: A.Extent of body change present B.Suddenness of the change C.Obviousness of the change D.Client’s perception of the change Answer: (D) Client’s perception of the change It is not reality, but the client’s feeling about the change that is the most important determinant of the ability to cope. The client should be encouraged to his feelings. 134. Larry is diagnosed as having myelocytic leukemia and is admitted to the hospital for chemotherapy. Larry discusses his recent diagnosis of leukemia by referring to statistical facts and figures. The nurse recognizes that Larry is using the defense mechanism known as: A.Reaction formation B.Sublimation C.Intellectualization D.Projection Answer: (C) Intellectualization People use defense mechanisms to cope with stressful events. Intellectualization is the use of reasoning and thought processes to avoid the emotional upsets. 135. The laboratory results of the client with leukemia indicate bone marrow depression. The nurse should encourage the client to: A.Increase his activity level and ambulate frequently B.Sleep with the head of his bed slightly elevated C.Drink citrus juices frequently for nourishment D.Use a soft toothbrush and electric razor Answer: (D) Use a soft toothbrush and electric razor Suppression of red bone marrow increases bleeding susceptibility associated with thrombocytopenia, decreased platelets. Anemia and leucopenia are the two other problems noted with bone marrow depression. 136. Dennis receives a blood transfusion and develops flank pain, chills, fever and hematuria. The nurse recognizes that Dennis is probably experiencing: A.An anaphylactic transfusion reaction B.An allergic transfusion reaction

C.A hemolytic transfusion reaction D.A pyrogenic transfusion reaction Answer: (C) A hemolytic transfusion reaction This results from a recipient’s antibodies that are incompatible with transfused RBC’s; also called type II hypersensitivity; these signs result from RBC hemolysis, agglutination, and capillary plugging that can damage renal function, thus the flank pain and hematuria and the other manifestations. 137. A client jokes about his leukemia even though he is becoming sicker and weaker. The nurse’s most therapeutic response would be: A.“Your laugher is a cover for your fear.” B.“He who laughs on the outside, cries on the inside.” C.“Why are you always laughing?” D.“Does it help you to joke about your illness?” Answer: (D) “Does it help you to joke about your illness?” This non-judgmentally on the part of the nurse points out the client’s behavior. 138. In dealing with a dying client who is in the denial stage of grief, the best nursing approach is to: A.Agree with and encourage the client’s denial B.Reassure the client that everything will be okay C.Allow the denial but be available to discuss death D.Leave the client alone to discuss the loss Answer: (C) Allow the denial but be available to discuss death This does not take away the client’s only way of coping, and it permits future movement through the grieving process when the client is ready. Dying clients move through the different stages of grieving and the nurse must be ready to intervene in all these stages. 139. During and 8 hour shift, Mario drinks two 6 oz. cups of tea and vomits 125 ml of fluid. During this 8 hour period, his fluid balance would be: A.+55 ml B.+137 ml C.+235 ml D.+485 ml Answer: (C) +235 ml The client’s intake was 360 ml (6oz x 30 ml) and loss was 125 ml of fluid; loss is subtracted from intake 140. Mr. Ong is admitted to the hospital with a diagnosis of Left-sided CHF. In the assessment, the nurse should expect to find: A.Crushing chest pain B.Dyspnea on exertion C.Extensive peripheral edema D.Jugular vein distention Answer: (B) Dyspnea on exertion Pulmonary congestion and edema occur because of fluid extravasation from the pulmonary capillary bed, resulting in difficult breathing. Left-sided heart failure creates a backward effect on the pulmonary system that leads to pulmonary congestion. 141. The physician orders on a client with CHF a cardiac glycoside, a vasodilator, and furosemide (Lasix). The nurse understands Lasix exerts is effects in the: A.Distal tubule B.Collecting duct C.Glomerulus of the nephron

D.Ascending limb of the loop of Henle Answer: (D) Ascending limb of the loop of Henle This is the site of action of Lasix being a potent loop diuretic. 142. Mr. Ong weighs 210 lbs on admission to the hospital. After 2 days of diuretic therapy he weighs 205.5 lbs. The nurse could estimate that the amount of fluid he has lost is: A.0.5 L B.1.0 L C.2.0 L D.3.5 L Answer: (C) 2.0 L One liter of fluid weighs approximately 2.2 lbs. Therefore a 4.5 lbs weight loss equals approximately 2 Liters. 143. Mr. Ong, a client with CHF, has been receiving a cardiac glycoside, a diuretic, and a vasodilator drug. His apical pulse rate is 44 and he is on bed rest. The nurse concludes that his pulse rate is most likely the result of the: A.Diuretic B.Vasodilator C.Bed-rest regimen D.Cardiac glycoside Answer: (D) Cardiac glycoside A cardiac glycoside such as digitalis increases force of cardiac contraction, decreases the conduction speed of impulses within the myocardium and slows the heart rate. 144. The diet ordered for a client with CHF permits him to have a 190 g of carbohydrates, 90 g of fat and 100 g of protein. The nurse understands that this diet contains approximately: A.2200 calories B.2000 calories C.2800 calories D.1600 calories Answer: (B) 2000 calories There are 9 calories in each gram of fat and 4 calories in each gram of carbohydrate and protein 145. After the acute phase of congestive heart failure, the nurse should expect the dietary management of the client to include the restriction of: A.Magnesium B.Sodium C.Potassium D.Calcium Answer: (B) Sodium Restriction of sodium reduces the amount of water retention that reduces the cardiac workload 146. Jude develops GI bleeding and is admitted to the hospital. An important etiologic clue for the nurse to explore while taking his history would be: A.The medications he has been taking B.Any recent foreign travel C.His usual dietary pattern D.His working patterns Answer: (A) The medications he has been taking Some medications, such as aspirin and prednisone, irritate the stomach lining and may cause bleeding with prolonged use

147. The meal pattern that would probably be most appropriate for a client recovering from GI bleeding is: A.Three large meals large enough to supply adequate energy. B.Regular meals and snacks to limit gastric discomfort C.Limited food and fluid intake when he has pain D.A flexible plan according to his appetite Answer: (B) Regular meals and snacks to limit gastric discomfort Presence of food in the stomach at regular intervals interacts with HCl limiting acid mucosal irritation. Mucosal irritation can lead to bleeding. 148. A client with a history of recurrent GI bleeding is admitted to the hospital for a gastrectomy. Following surgery, the client has a nasogastric tube to low continuous suction. He begins to hyperventilate. The nurse should be aware that this pattern will alter his arterial blood gases by: A.Increasing HCO3 B.Decreasing PCO2 C.Decreasing pH D.Decreasing PO2 Answer: (B) Decreasing PCO2 Hyperventilation results in the increased elimination of carbon dioxide from the blood that can lead to respiratory alkalosis. 149. Routine postoperative IV fluids are designed to supply hydration and electrolyte and only limited energy. Because 1 L of a 5% dextrose solution contains 50 g of sugar, 3 L per day would apply approximately: A.400 Kilocalories B.600 Kilocalories C.800 Kilocalories D.1000 Kilocalories Answer: (B) 600 Kilocalories Carbohydrates provide 4 kcal/ gram; therefore 3L x 50 g/L x 4 kcal/g = 600 kcal; only about a third of the basal energy need. 150. Thrombus formation is a danger for all postoperative clients. The nurse should act independently to prevent this complication by: A.Encouraging adequate fluids B.Applying elastic stockings C.Massaging gently the legs with lotion D.Performing active-assistive leg exercises Answer: (D) Performing active-assistive leg exercises Inactivity causes venous stasis, hypercoagulability, and external pressure against the veins, all of which lead to thrombus formation. Early ambulation or exercise of the lower extremities reduces the occurrence of this phenomenon 151. An unconscious client is admitted to the ICU, IV fluids are started and a Foley catheter is inserted. With an indwelling catheter, urinary infection is a potential danger. The nurse can best plan to avoid this problem by: A.Emptying the drainage bag frequently B.Collecting a weekly urine specimen C.Maintaining the ordered hydration D.Assessing urine specific gravity Answer: (C) Maintaining the ordered hydration Promoting hydration, maintains urine production at a higher rate, which flushes the bladder and prevents urinary stasis and possible infection

152. The nurse performs full range of motion on a bedridden client’s extremities. When putting his ankle through range of motion, the nurse must perform: A.Flexion, extension and left and right rotation B.Abduction, flexion, adduction and extension C.Pronation, supination, rotation, and extension D.Dorsiflexion, plantar flexion, eversion and inversion Answer: (D) Dorsiflexion, plantar flexion, eversion and inversion These movements include all possible range of motion for the ankle joint 153. A client has been in a coma for 2 months. The nurse understands that to prevent the effects of shearing force on the skin, the head of the bed should be at an angle of: A.30 degrees B.45 degrees C.60 degrees D.90 degrees Answer: (A) 30 degrees Shearing force occurs when 2 surfaces move against each other; when the bed is at an angle greater than 30 degrees, the torso tends to slide and causes this phenomenon. Shearing forces are good contributory factors of pressure sores. 154. Rene, age 62, is scheduled for a TURP after being diagnosed with a Benign Prostatic Hyperplasia (BPH). As part of the preoperative teaching, the nurse should tell the client that after surgery: A.Urinary control may be permanently lost to some degree B.Urinary drainage will be dependent on a urethral catheter for 24 hours C.Frequency and burning on urination will last while the cystotomy tube is in place D.His ability to perform sexually will be permanently impaired Answer: (B) Urinary drainage will be dependent on a urethral catheter for 24 hours An indwelling urethral catheter is used, because surgical trauma can cause urinary retention leading to further complications such as bleeding. 155. The transurethral resection of the prostate is performed on a client with BPH. Following surgery, nursing care should include: A.Changing the abdominal dressing B.Maintaining patency of the cystotomy tube C.Maintaining patency of a three-way Foley catheter for cystoclysis D.Observing for hemorrhage and wound infection Answer: (C) Maintaining patency of a three-way Foley catheter for cystoclysis Patency of the catheter promotes bladder decompression, which prevents distention and bleeding. Continuous flow of fluid through the bladder limits clot formation and promotes hemostasis 156. In the early postoperative period following a transurethral surgery, the most common complication the nurse should observe for is: A.Sepsis B.Hemorrhage C.Leakage around the catheter D.Urinary retention with overflow Answer: (B) Hemorrhage After transurethral surgery, hemorrhage is common because of venous oozing and bleeding from many small arteries in the prostatic bed.

157. Following prostate surgery, the retention catheter is secured to the client’s leg causing slight traction of the inflatable balloon against the prostatic fossa. This is done to: A.Limit discomfort B.Provide hemostasis C.Reduce bladder spasms D.Promote urinary drainage Answer: (B) Provide hemostasis The pressure of the balloon against the small blood vessels of the prostate creates a tampon-like effect that causes them to constrict thereby preventing bleeding. 158. Twenty-four hours after TURP surgery, the client tells the nurse he has lower abdominal discomfort. The nurse notes that the catheter drainage has stopped. The nurse’s initial action should be to: A.Irrigate the catheter with saline B.Milk the catheter tubing C.Remove the catheter D.Notify the physician Answer: (B) Milk the catheter tubing Milking the tubing will usually dislodge the plug and will not harm the client. A physician’s order is not necessary for a nurse to check catheter patency. 159. The nurse would know that a post-TURP client understood his discharge teaching when he says “I should:” A.Get out of bed into a chair for several hours daily B.Call the physician if my urinary stream decreases C.Attempt to void every 3 hours when I’m awake D.Avoid vigorous exercise for 6 months after surgery Answer: (B) Call the physician if my urinary stream decreases Urethral mucosa in the prostatic area is destroyed during surgery and strictures my form with healing that causes partial or even complete ueinary obstruction. 160. Lucy is admitted to the surgical unit for a subtotal thyroidectomy. She is diagnosed with Grave’s Disease. When assessing Lucy, the nurse would expect to find: A.Lethargy, weight gain, and forgetfulness B.Weight loss, protruding eyeballs, and lethargy C.Weight loss, exopthalmos and restlessness D.Constipation, dry skin, and weight gain Answer: (C) Weight loss, exopthalmos and restlessness Classic signs associated with hyperthyroidism are weight loss and restlessness because of increased basal metabolic rate. Exopthalmos is due to peribulbar edema. 161. Lucy undergoes Subtotal Thyroidectomy for Grave’s Disease. In planning for the client’s return from the OR, the nurse would consider that in a subtotal thyroidectomy: A.The entire thyroid gland is removed B.A small part of the gland is left intact C.One parathyroid gland is also removed D.A portion of the thyroid and four parathyroids are removed Answer: (B) A small part of the gland is left intact Remaining thyroid tissue may provide enough hormone for normal function. Total thyroidectomy is generally done in clients with Thyroid Ca. 162. Before a post- thyroidectomy client returns to her room from the OR, the nurse plans to set up emergency equipment, which should include:

A.A crash cart with bed board B.A tracheostomy set and oxygen C.An airway and rebreathing mask D.Two ampules of sodium bicarbonate Answer: (B) A tracheostomy set and oxygen Acute respiratory obstruction in the post-operative period can result from edema, subcutaneous bleeding that presses on the trachea, nerve damage, or tetany. 163. When a post-thyroidectomy client returns from surgery the nurse assesses her for unilateral injury of the laryngeal nerve every 30 to 60 minutes by: A.Observing for signs of tetany B.Checking her throat for swelling C.Asking her to state her name out loud D.Palpating the side of her neck for blood seepage Answer: (C) Asking her to state her name out loud If the recurrent laryngeal nerve is damaged during surgery, the client will be hoarse and have difficult speaking. 164. On a post-thyroidectomy client’s discharge, the nurse teaches her to observe for signs of surgically induced hypothyroidism. The nurse would know that the client understands the teaching when she states she should notify the physician if she develops: A.Intolerance to heat B.Dry skin and fatigue C.Progressive weight loss D.Insomnia and excitability Answer: (B) Dry skin and fatigue Dry skin is most likely caused by decreased glandular function and fatigue caused by decreased metabolic rate. Body functions and metabolism are decreased in hypothyroidism. 165. A client’s exopthalmos continues inspite of thyroidectomy for Grave’s Disease. The nurse teaches her how to reduce discomfort and prevent corneal ulceration. The nurse recognizes that the client understands the teaching when she says: “I should: A.Elevate the head of my bed at night B.Avoid moving my extra-ocular muscles C.Avoid using a sleeping mask at night D.Avoid excessive blinking Answer: (C) Avoid using a sleeping mask at night The mask may irritate or scratch the eye if the client turns and lies on it during the night. 166. Clara is a 37-year old cook. She is admitted for treatment of partial and full-thickness burns of her entire right lower extremity and the anterior portion of her right upper extremity. Her respiratory status is compromised, and she is in pain and anxious. Performing an immediate appraisal, using the rule of nines, the nurse estimates the percent of Clara’s body surface that is burned is: A.4.5% B.9% C.18 % D.22.5% Answer: (D) 22.5% The entire right lower extremity is 18% the anterior portion of the right upper extremity is 4.5% giving a total of 22.5%

167. The nurse applies mafenide acetate (Sulfamylon cream) to Clara, who has second and third degree burns on the right upper and lower extremities, as ordered by the physician. This medication will: A.Inhibit bacterial growth B.Relieve pain from the burn C.Prevent scar tissue formation D.Provide chemical debridement Answer: (A) Inhibit bacterial growth Sulfamylon is effective against a wide variety of gram positive and gram negative organisms including anaerobes 168. Forty-eight hours after a burn injury, the physician orders for the client 2 liters of IV fluid to be administered q12 h. The drop factor of the tubing is 10 gtt/ml. The nurse should set the flow to provide: A.18 gtt/min B.28 gtt/min C.32 gtt/min D.36 gtt/min Answer: (B) 28 gtt/min This is the correct flow rate; multiply the amount to be infused (2000 ml) by the drop factor (10) and divide the result by the amount of time in minutes (12 hours x 60 minutes) 169. Clara, a burn client, receives a temporary heterograft (pig skin) on some of her burns. These grafts will: A.Debride necrotic epithelium B.Be sutured in place for better adherence C.Relieve pain and promote rapid epithelialization D.Frequently be used concurrently with topical antimicrobials. Answer: (C) Relieve pain and promote rapid epithelialization The graft covers nerve endings, which reduces pain and provides a framework for granulation that promotes effective healing. 170. A client with burns on the chest has periodic episodes of dyspnea. The position that would provide for the greatest respiratory capacity would be the: A.Semi-fowler’s position B.Sims’ position C.Orthopneic position D.Supine position Answer: (C) Orthopneic position The orthopneic position lowers the diaphragm and provides for maximal thoracic expansion 171. Jane, a 20- year old college student is admiited to the hospital with a tentative diagnosis of myasthenia gravis. She is scheduled to have a series of diagnostic studies for myasthenia gravis, including a Tensilon test. In preparing her for this procedure, the nurse explains that her response to the medication will confirm the diagnosis if Tensilon produces: A.Brief exaggeration of symptoms B.Prolonged symptomatic improvement C.Rapid but brief symptomatic improvement D.Symptomatic improvement of just the ptosis Answer: (C) Rapid but brief symptomatic improvement Tensilon acts systemically to increase muscle strength; with a peak effect in 30 seconds, It lasts several minutes. 172. The initial nursing goal for a client with myasthenia gravis during the diagnostic phase of her hospitalization would be to: A.Develop a teaching plan B.Facilitate psychologic adjustment

C.Maintain the present muscle strength D.Prepare for the appearance of myasthenic crisis Answer: (C) Maintain the present muscle strength Until diagnosis is confirmed, primary goal should be to maintain adequate activity and prevent muscle atrophy 173. The most significant initial nursing observations that need to be made about a client with myasthenia include: A.Ability to chew and speak distinctly B.Degree of anxiety about her diagnosis C.Ability to smile an to close her eyelids D.Respiratory exchange and ability to swallow Answer: (D) Respiratory exchange and ability to swallow Muscle weakness can lead to respiratory failure that will require emergency intervention and inability to swallow may lead to aspiration 174. Helen is diagnosed with myasthenia gravis and pyridostigmine bromide (Mestinon) therapy is started. The Mestinon dosage is frequently changed during the first week. While the dosage is being adjusted, the nurse’s priority intervention is to: A.Administer the medication exactly on time B.Administer the medication with food or mild C.Evaluate the client’s muscle strength hourly after medication D.Evaluate the client’s emotional side effects between doses Answer: (C) Evaluate the client’s muscle strength hourly after medication Peak response occurs 1 hour after administration and lasts up to 8 hours; the response will influence dosage levels. 175. Helen, a client with myasthenia gravis, begins to experience increased difficulty in swallowing. To prevent aspiration of food, the nursing action that would be most effective would be to: A.Change her diet order from soft foods to clear liquids B.Place an emergency tracheostomy set in her room C.Assess her respiratory status before and after meals D.Coordinate her meal schedule with the peak effect of her medication, Mestinon Answer: (D) Coordinate her meal schedule with the peak effect of her medication, Mestinon Dysphagia should be minimized during peak effect of Mestinon, thereby decreasing the probability of aspiration. Mestinon can increase her muscle strength including her ability to swallow.

Sponsor Documents

Or use your account on DocShare.tips

Hide

Forgot your password?

Or register your new account on DocShare.tips

Hide

Lost your password? Please enter your email address. You will receive a link to create a new password.

Back to log-in

Close